LSAT and Law School Admissions Forum

Get expert LSAT preparation and law school admissions advice from PowerScore Test Preparation.

 Administrator
PowerScore Staff
  • PowerScore Staff
  • Posts: 8917
  • Joined: Feb 02, 2011
|
#85873
Complete Question Explanation

The correct answer choice is (D).

Answer choice (A):

Answer choice (B):

Answer choice (C):

Answer choice (D): This is the correct answer choice.

Answer choice (E):

This explanation is still in progress. Please post any questions below!
 vangorgc
  • Posts: 12
  • Joined: Nov 14, 2020
|
#83496
Hi PowerScore,
With this one, I see how C is wrong but I don't see how D might also be wrong. The last paragraph says, "in support of the asteroid-impact model, it has been argued that the gradual cooling of the average ocean temperature would enable progressively smaller astroid impacts (which are known to occur more frequently than larger impacts) to cool the earth's climate sufficiently to induce ice-cap growth and reversals." At the pace I was going, I took this to mean that ice caps could cause changes in ocean circulation and therefore D could be ruled out.
I was between D and C on this. Did I just overthink D? I thought it was trying to play a trick and that the blobs played another role.
Grace
 Adam Tyson
PowerScore Staff
  • PowerScore Staff
  • Posts: 5153
  • Joined: Apr 14, 2011
|
#83531
Answer D was what we sometimes call a "shell game" answer, Grace. That's where the authors take things from the passage and scramble them up in ways that the passage did not provide. In the passage, circulation was discussed, but it was circulation of the outer core fluid that mattered. Ocean temperatures were discussed as possibly contributing to reversals in polarity by making it easier for smaller asteroids to trigger the growth of the ice caps, but ocean circulation patterns were never mentioned. Since we are supposed to pick the one thing that was NOT mentioned as a cause, D is our winner!
User avatar
 mab9178
  • Posts: 96
  • Joined: May 02, 2022
|
#99473
Hi,

Lines 45, specifically the wording "redistributing the Earth's seawater," gave me a long pause as to whether D is the "cannot be true" answer choice; how can there be a "redistribution of the Earth's seawater" without "changes in circulation patterns in the Earth's oceans," at least for a short period of time?

Or maybe this is a multi-causation observation in the following way:

yeah , sure you cannot have redistribution of seawater water without changes in circulation patterns in the Earth's atmosphere; BUT, according to the "asteroid impact theory," the redistribution of the seawater also caused increases in the rotational acceleration of the mantle, and it is this - i.e. the increase in the rotational acceleration of the mantle - that this theory argues triggered the chain causing the reversal, and NOT the changes in circulation patterns in the Earth's ocean though it too had to hav occurred by the redistribution of seawater.

In other words, multi-causation because: the redistribution caused both changes in circulation patterns in the Earth's ocean AND the increase in the acceleration of the mantle, but only the latter, according to the second theory, causes the reversal; and since the question is caused the reversal EXCEPT, the latter (changes in circulation patterns in the Earth's ocean) the correct, cannot be true, answer-choice.

Or, am I off target?

Thank you
Mazen
User avatar
 Jeff Wren
PowerScore Staff
  • PowerScore Staff
  • Posts: 389
  • Joined: Oct 19, 2022
|
#99905
Hi Mazen,

I can understand why you might think that "redistributing the Earth's seawater" would cause "changes in circulation patterns in the Earth's oceans," but "circulation patterns in the Earth's oceans" were never discussed in the passage, and we can't assume anything about them here.

What makes this answer especially tricky is that "circulation patterns" were mentioned in passage, but not in the context of the Earth's oceans. The "heat circulation pattern of the outer core fluid" is discussed in lines 27-28 and 38.

Unlike Answer D, the other four answers were specifically mentioned in the passage, so it may be easier to arrive at Answer D by eliminating the other four wrong answers here.

Get the most out of your LSAT Prep Plus subscription.

Analyze and track your performance with our Testing and Analytics Package.